Why is the scalar product of two four-vectors Lorentz-invariant?











up vote
4
down vote

favorite












Why is the scalar product of two four-vectors Lorentz-invariant?



For instance, given two four-vector $A^mu$ and $B^mu$, so their scalar product is $Acdot B=A^mu B_mu=A^mu g_{munu}B^{nu}$.



Why is $A^mu g_{munu}B^{nu}$ Lorentz-invariant?










share|cite|improve this question




















  • 3




    Have a look at eranreches' answer to Scalar invariance under Lorentz-transformation. That should answer your question or at least help you understand it.
    – John Rennie
    Nov 20 at 11:25












  • That's the only way the speed of light can be the same in all frames. Any standard reference on special relativity should explain this. I personally like the book "Introduction to Special Relativity" by Rindler.
    – Eric David Kramer
    Nov 20 at 12:14










  • A scalar product like $A^mu B_mu$ is invariant under any smooth coordinate transformation. The only thing that's special about Lorentz transformations is that they leave the components of the metric invariant.
    – Ben Crowell
    Nov 20 at 15:00










  • Possible duplicate of Lorentz invariance of the Minkowski metric
    – AccidentalFourierTransform
    Nov 20 at 18:52















up vote
4
down vote

favorite












Why is the scalar product of two four-vectors Lorentz-invariant?



For instance, given two four-vector $A^mu$ and $B^mu$, so their scalar product is $Acdot B=A^mu B_mu=A^mu g_{munu}B^{nu}$.



Why is $A^mu g_{munu}B^{nu}$ Lorentz-invariant?










share|cite|improve this question




















  • 3




    Have a look at eranreches' answer to Scalar invariance under Lorentz-transformation. That should answer your question or at least help you understand it.
    – John Rennie
    Nov 20 at 11:25












  • That's the only way the speed of light can be the same in all frames. Any standard reference on special relativity should explain this. I personally like the book "Introduction to Special Relativity" by Rindler.
    – Eric David Kramer
    Nov 20 at 12:14










  • A scalar product like $A^mu B_mu$ is invariant under any smooth coordinate transformation. The only thing that's special about Lorentz transformations is that they leave the components of the metric invariant.
    – Ben Crowell
    Nov 20 at 15:00










  • Possible duplicate of Lorentz invariance of the Minkowski metric
    – AccidentalFourierTransform
    Nov 20 at 18:52













up vote
4
down vote

favorite









up vote
4
down vote

favorite











Why is the scalar product of two four-vectors Lorentz-invariant?



For instance, given two four-vector $A^mu$ and $B^mu$, so their scalar product is $Acdot B=A^mu B_mu=A^mu g_{munu}B^{nu}$.



Why is $A^mu g_{munu}B^{nu}$ Lorentz-invariant?










share|cite|improve this question















Why is the scalar product of two four-vectors Lorentz-invariant?



For instance, given two four-vector $A^mu$ and $B^mu$, so their scalar product is $Acdot B=A^mu B_mu=A^mu g_{munu}B^{nu}$.



Why is $A^mu g_{munu}B^{nu}$ Lorentz-invariant?







special-relativity metric-tensor tensor-calculus lorentz-symmetry invariants






share|cite|improve this question















share|cite|improve this question













share|cite|improve this question




share|cite|improve this question








edited Nov 20 at 13:20









Emilio Pisanty

81.3k21194400




81.3k21194400










asked Nov 20 at 10:26









A.Luo

858




858








  • 3




    Have a look at eranreches' answer to Scalar invariance under Lorentz-transformation. That should answer your question or at least help you understand it.
    – John Rennie
    Nov 20 at 11:25












  • That's the only way the speed of light can be the same in all frames. Any standard reference on special relativity should explain this. I personally like the book "Introduction to Special Relativity" by Rindler.
    – Eric David Kramer
    Nov 20 at 12:14










  • A scalar product like $A^mu B_mu$ is invariant under any smooth coordinate transformation. The only thing that's special about Lorentz transformations is that they leave the components of the metric invariant.
    – Ben Crowell
    Nov 20 at 15:00










  • Possible duplicate of Lorentz invariance of the Minkowski metric
    – AccidentalFourierTransform
    Nov 20 at 18:52














  • 3




    Have a look at eranreches' answer to Scalar invariance under Lorentz-transformation. That should answer your question or at least help you understand it.
    – John Rennie
    Nov 20 at 11:25












  • That's the only way the speed of light can be the same in all frames. Any standard reference on special relativity should explain this. I personally like the book "Introduction to Special Relativity" by Rindler.
    – Eric David Kramer
    Nov 20 at 12:14










  • A scalar product like $A^mu B_mu$ is invariant under any smooth coordinate transformation. The only thing that's special about Lorentz transformations is that they leave the components of the metric invariant.
    – Ben Crowell
    Nov 20 at 15:00










  • Possible duplicate of Lorentz invariance of the Minkowski metric
    – AccidentalFourierTransform
    Nov 20 at 18:52








3




3




Have a look at eranreches' answer to Scalar invariance under Lorentz-transformation. That should answer your question or at least help you understand it.
– John Rennie
Nov 20 at 11:25






Have a look at eranreches' answer to Scalar invariance under Lorentz-transformation. That should answer your question or at least help you understand it.
– John Rennie
Nov 20 at 11:25














That's the only way the speed of light can be the same in all frames. Any standard reference on special relativity should explain this. I personally like the book "Introduction to Special Relativity" by Rindler.
– Eric David Kramer
Nov 20 at 12:14




That's the only way the speed of light can be the same in all frames. Any standard reference on special relativity should explain this. I personally like the book "Introduction to Special Relativity" by Rindler.
– Eric David Kramer
Nov 20 at 12:14












A scalar product like $A^mu B_mu$ is invariant under any smooth coordinate transformation. The only thing that's special about Lorentz transformations is that they leave the components of the metric invariant.
– Ben Crowell
Nov 20 at 15:00




A scalar product like $A^mu B_mu$ is invariant under any smooth coordinate transformation. The only thing that's special about Lorentz transformations is that they leave the components of the metric invariant.
– Ben Crowell
Nov 20 at 15:00












Possible duplicate of Lorentz invariance of the Minkowski metric
– AccidentalFourierTransform
Nov 20 at 18:52




Possible duplicate of Lorentz invariance of the Minkowski metric
– AccidentalFourierTransform
Nov 20 at 18:52










3 Answers
3






active

oldest

votes

















up vote
12
down vote



accepted










Frankly, you're looking at this backwards.




Why is $A^mu g_{munu}B^{nu}$ Lorentz-invariant?




That's the wrong way around: $A^mu g_{munu}B^{nu}$ is Lorentz invariant because Lorentz transformations are defined as the class of transformations that leaves $A^mu g_{munu}B^{nu}$ invariant.



Generally, if you transform $A^mu$ and $B^mu$ by some linear transformation with the transformation matrix $Lambda^mu_{ nu}$, then their transformed values will be $tilde A^mu = Lambda^mu_{ nu}A^nu$ and $tilde B^mu = Lambda^mu_{ nu} B^nu$ (using Einstein summations). This means that the transformed inner product will be
begin{align}
tilde A^mu g_{munu} tilde B^{nu}
& =
(Lambda^mu_{ alpha}A^alpha) g_{munu} (Lambda^nu_{ beta}B^beta)
\ & =
A^alpha (Lambda^mu_{ alpha} g_{munu} Lambda^nu_{ beta} )B^beta
\ & =
A^mu (Lambda^gamma_{ mu} g_{gammadelta} Lambda^delta_{ nu} )B^nu
qquadqquadtext{(by re-labelling)}
\ & stackrel{text{required}}=
A^mu g_{munu} B^nu.
end{align}

Thus, for $A^mu g_{munu} B^nu$ to be invariant we require that
$$
A^mu (Lambda^gamma_{ mu} g_{gammadelta} Lambda^delta_{ nu} )B^nu
=
A^mu g_{munu} B^nu
$$

for all $A^mu$ and $B^mu$, and by judicious choices of those vectors (basically running each independently over the basis in use) that can only be the case if
$$
Lambda^gamma_{ mu} g_{gammadelta} Lambda^delta_{ nu}
=
g_{munu},
$$

which forms the core requirement on $Lambda^mu_{ nu}$ for it to be a Lorentz transformation.






share|cite|improve this answer




























    up vote
    2
    down vote













    Here's the way to think about this -- why is the standard Euclidean dot product, $sum x_iy_i$ interesting? Well, it is interesting primarily from the perspective of rotations, due to the fact that rotations leave dot products invariant. The reason this is so is that this dot product can be written as $|x||y|cosDeltatheta$, and rotations leave magnitudes and relative angles invariant.



    Is the standard Euclidean norm $|x|$ invariant under Lorentz transformations? Of course not -- for instance, $Delta t^2+Delta x^2$ is clearly not invariant, but $Delta t^2-Delta x^2$ is. Similarly, $E^2+p^2$ is not important, but $E^2-p^2$ is. The reason this is the case is that Lorentz boosts are fundamentally skew transformations, which means the invariant locus is a hyperbola, not a circle. So you have $cosh^2 xi - sinh^2 xi = 1$, and $x_0^2-x_1^2$ is the right way to think of the norm on Minkowski space.



    Similarly, Lorentz boosts change the rapidity $xi$ by a simple displacement, so $Delta xi$ is invariant. From this point, it's a simple exercise to show that



    $$|x||y|coshxi=x_0y_0-x_1y_1$$



    (as for the remaining dimensions -- remember that the standard Euclidean dot product is still relevant in space, so you just need to write $x_0y_0-xcdot y=x_0y_0-x_1y_1-x_2y_2-x_3y_3$.)






    share|cite|improve this answer




























      up vote
      0
      down vote













      A vector $mathbf{v} = v^i , mathbf{e}_i = q^j ,mathbf{u}_j$ has different vector components ($v^i$, $q^j$ in this case) in different bases (${mathbf{e}}$,${mathbf{u}}$, in our example) which we can interpret as different reference frames (different axes with different origins).



      Physicists are lazy: they refer to the vector components $v^i$ as vectors, which is a misnomer! A true vector $mathbf{v}$ exists out width whichever basis you choose to work in but to know its entries you must reference these with respect to a given basis: this is just elementary linear algebra.



      Now, the magnitude of a vector is independent of whichever bases you choose for its description (that is, geometrically speaking its length is fixed):



      $$
      v^2 = v^i ; v_i , (mathbf{e}^i cdot mathbf{e}_i) = q^{j} , q_j,(mathbf{u}_jcdot mathbf{u}^j)
      .
      tag{assuming orthonormal bases}
      $$



      Hence, scalars do not transform upon of change of basis. In fact it doesn't make much sense to talk about basis for scalars since intuitively these are just numbers.



      However, one other way to look at this is to consider a scalar a special type of vector with only one entry and one orthonormal basis (the number 1): its "length" must also be fixed. Hence, this one-dimensional "vector" is the same independent of reference frame.



      This is true for all vectors, including special relativistic four-vectors.



      As a sanity check, one of the tenets of special relativity is that $c$, the speed of light and a scalar, is the same for all observers. This could not be so if it was somehow different in different frames.






      share|cite|improve this answer























      • @EmilioPisanty Yes, indeed! Fixed.
        – OldTomMorris
        Nov 20 at 21:59











      Your Answer





      StackExchange.ifUsing("editor", function () {
      return StackExchange.using("mathjaxEditing", function () {
      StackExchange.MarkdownEditor.creationCallbacks.add(function (editor, postfix) {
      StackExchange.mathjaxEditing.prepareWmdForMathJax(editor, postfix, [["$", "$"], ["\\(","\\)"]]);
      });
      });
      }, "mathjax-editing");

      StackExchange.ready(function() {
      var channelOptions = {
      tags: "".split(" "),
      id: "151"
      };
      initTagRenderer("".split(" "), "".split(" "), channelOptions);

      StackExchange.using("externalEditor", function() {
      // Have to fire editor after snippets, if snippets enabled
      if (StackExchange.settings.snippets.snippetsEnabled) {
      StackExchange.using("snippets", function() {
      createEditor();
      });
      }
      else {
      createEditor();
      }
      });

      function createEditor() {
      StackExchange.prepareEditor({
      heartbeatType: 'answer',
      convertImagesToLinks: false,
      noModals: true,
      showLowRepImageUploadWarning: true,
      reputationToPostImages: null,
      bindNavPrevention: true,
      postfix: "",
      imageUploader: {
      brandingHtml: "Powered by u003ca class="icon-imgur-white" href="https://imgur.com/"u003eu003c/au003e",
      contentPolicyHtml: "User contributions licensed under u003ca href="https://creativecommons.org/licenses/by-sa/3.0/"u003ecc by-sa 3.0 with attribution requiredu003c/au003e u003ca href="https://stackoverflow.com/legal/content-policy"u003e(content policy)u003c/au003e",
      allowUrls: true
      },
      noCode: true, onDemand: true,
      discardSelector: ".discard-answer"
      ,immediatelyShowMarkdownHelp:true
      });


      }
      });














      draft saved

      draft discarded


















      StackExchange.ready(
      function () {
      StackExchange.openid.initPostLogin('.new-post-login', 'https%3a%2f%2fphysics.stackexchange.com%2fquestions%2f442119%2fwhy-is-the-scalar-product-of-two-four-vectors-lorentz-invariant%23new-answer', 'question_page');
      }
      );

      Post as a guest















      Required, but never shown

























      3 Answers
      3






      active

      oldest

      votes








      3 Answers
      3






      active

      oldest

      votes









      active

      oldest

      votes






      active

      oldest

      votes








      up vote
      12
      down vote



      accepted










      Frankly, you're looking at this backwards.




      Why is $A^mu g_{munu}B^{nu}$ Lorentz-invariant?




      That's the wrong way around: $A^mu g_{munu}B^{nu}$ is Lorentz invariant because Lorentz transformations are defined as the class of transformations that leaves $A^mu g_{munu}B^{nu}$ invariant.



      Generally, if you transform $A^mu$ and $B^mu$ by some linear transformation with the transformation matrix $Lambda^mu_{ nu}$, then their transformed values will be $tilde A^mu = Lambda^mu_{ nu}A^nu$ and $tilde B^mu = Lambda^mu_{ nu} B^nu$ (using Einstein summations). This means that the transformed inner product will be
      begin{align}
      tilde A^mu g_{munu} tilde B^{nu}
      & =
      (Lambda^mu_{ alpha}A^alpha) g_{munu} (Lambda^nu_{ beta}B^beta)
      \ & =
      A^alpha (Lambda^mu_{ alpha} g_{munu} Lambda^nu_{ beta} )B^beta
      \ & =
      A^mu (Lambda^gamma_{ mu} g_{gammadelta} Lambda^delta_{ nu} )B^nu
      qquadqquadtext{(by re-labelling)}
      \ & stackrel{text{required}}=
      A^mu g_{munu} B^nu.
      end{align}

      Thus, for $A^mu g_{munu} B^nu$ to be invariant we require that
      $$
      A^mu (Lambda^gamma_{ mu} g_{gammadelta} Lambda^delta_{ nu} )B^nu
      =
      A^mu g_{munu} B^nu
      $$

      for all $A^mu$ and $B^mu$, and by judicious choices of those vectors (basically running each independently over the basis in use) that can only be the case if
      $$
      Lambda^gamma_{ mu} g_{gammadelta} Lambda^delta_{ nu}
      =
      g_{munu},
      $$

      which forms the core requirement on $Lambda^mu_{ nu}$ for it to be a Lorentz transformation.






      share|cite|improve this answer

























        up vote
        12
        down vote



        accepted










        Frankly, you're looking at this backwards.




        Why is $A^mu g_{munu}B^{nu}$ Lorentz-invariant?




        That's the wrong way around: $A^mu g_{munu}B^{nu}$ is Lorentz invariant because Lorentz transformations are defined as the class of transformations that leaves $A^mu g_{munu}B^{nu}$ invariant.



        Generally, if you transform $A^mu$ and $B^mu$ by some linear transformation with the transformation matrix $Lambda^mu_{ nu}$, then their transformed values will be $tilde A^mu = Lambda^mu_{ nu}A^nu$ and $tilde B^mu = Lambda^mu_{ nu} B^nu$ (using Einstein summations). This means that the transformed inner product will be
        begin{align}
        tilde A^mu g_{munu} tilde B^{nu}
        & =
        (Lambda^mu_{ alpha}A^alpha) g_{munu} (Lambda^nu_{ beta}B^beta)
        \ & =
        A^alpha (Lambda^mu_{ alpha} g_{munu} Lambda^nu_{ beta} )B^beta
        \ & =
        A^mu (Lambda^gamma_{ mu} g_{gammadelta} Lambda^delta_{ nu} )B^nu
        qquadqquadtext{(by re-labelling)}
        \ & stackrel{text{required}}=
        A^mu g_{munu} B^nu.
        end{align}

        Thus, for $A^mu g_{munu} B^nu$ to be invariant we require that
        $$
        A^mu (Lambda^gamma_{ mu} g_{gammadelta} Lambda^delta_{ nu} )B^nu
        =
        A^mu g_{munu} B^nu
        $$

        for all $A^mu$ and $B^mu$, and by judicious choices of those vectors (basically running each independently over the basis in use) that can only be the case if
        $$
        Lambda^gamma_{ mu} g_{gammadelta} Lambda^delta_{ nu}
        =
        g_{munu},
        $$

        which forms the core requirement on $Lambda^mu_{ nu}$ for it to be a Lorentz transformation.






        share|cite|improve this answer























          up vote
          12
          down vote



          accepted







          up vote
          12
          down vote



          accepted






          Frankly, you're looking at this backwards.




          Why is $A^mu g_{munu}B^{nu}$ Lorentz-invariant?




          That's the wrong way around: $A^mu g_{munu}B^{nu}$ is Lorentz invariant because Lorentz transformations are defined as the class of transformations that leaves $A^mu g_{munu}B^{nu}$ invariant.



          Generally, if you transform $A^mu$ and $B^mu$ by some linear transformation with the transformation matrix $Lambda^mu_{ nu}$, then their transformed values will be $tilde A^mu = Lambda^mu_{ nu}A^nu$ and $tilde B^mu = Lambda^mu_{ nu} B^nu$ (using Einstein summations). This means that the transformed inner product will be
          begin{align}
          tilde A^mu g_{munu} tilde B^{nu}
          & =
          (Lambda^mu_{ alpha}A^alpha) g_{munu} (Lambda^nu_{ beta}B^beta)
          \ & =
          A^alpha (Lambda^mu_{ alpha} g_{munu} Lambda^nu_{ beta} )B^beta
          \ & =
          A^mu (Lambda^gamma_{ mu} g_{gammadelta} Lambda^delta_{ nu} )B^nu
          qquadqquadtext{(by re-labelling)}
          \ & stackrel{text{required}}=
          A^mu g_{munu} B^nu.
          end{align}

          Thus, for $A^mu g_{munu} B^nu$ to be invariant we require that
          $$
          A^mu (Lambda^gamma_{ mu} g_{gammadelta} Lambda^delta_{ nu} )B^nu
          =
          A^mu g_{munu} B^nu
          $$

          for all $A^mu$ and $B^mu$, and by judicious choices of those vectors (basically running each independently over the basis in use) that can only be the case if
          $$
          Lambda^gamma_{ mu} g_{gammadelta} Lambda^delta_{ nu}
          =
          g_{munu},
          $$

          which forms the core requirement on $Lambda^mu_{ nu}$ for it to be a Lorentz transformation.






          share|cite|improve this answer












          Frankly, you're looking at this backwards.




          Why is $A^mu g_{munu}B^{nu}$ Lorentz-invariant?




          That's the wrong way around: $A^mu g_{munu}B^{nu}$ is Lorentz invariant because Lorentz transformations are defined as the class of transformations that leaves $A^mu g_{munu}B^{nu}$ invariant.



          Generally, if you transform $A^mu$ and $B^mu$ by some linear transformation with the transformation matrix $Lambda^mu_{ nu}$, then their transformed values will be $tilde A^mu = Lambda^mu_{ nu}A^nu$ and $tilde B^mu = Lambda^mu_{ nu} B^nu$ (using Einstein summations). This means that the transformed inner product will be
          begin{align}
          tilde A^mu g_{munu} tilde B^{nu}
          & =
          (Lambda^mu_{ alpha}A^alpha) g_{munu} (Lambda^nu_{ beta}B^beta)
          \ & =
          A^alpha (Lambda^mu_{ alpha} g_{munu} Lambda^nu_{ beta} )B^beta
          \ & =
          A^mu (Lambda^gamma_{ mu} g_{gammadelta} Lambda^delta_{ nu} )B^nu
          qquadqquadtext{(by re-labelling)}
          \ & stackrel{text{required}}=
          A^mu g_{munu} B^nu.
          end{align}

          Thus, for $A^mu g_{munu} B^nu$ to be invariant we require that
          $$
          A^mu (Lambda^gamma_{ mu} g_{gammadelta} Lambda^delta_{ nu} )B^nu
          =
          A^mu g_{munu} B^nu
          $$

          for all $A^mu$ and $B^mu$, and by judicious choices of those vectors (basically running each independently over the basis in use) that can only be the case if
          $$
          Lambda^gamma_{ mu} g_{gammadelta} Lambda^delta_{ nu}
          =
          g_{munu},
          $$

          which forms the core requirement on $Lambda^mu_{ nu}$ for it to be a Lorentz transformation.







          share|cite|improve this answer












          share|cite|improve this answer



          share|cite|improve this answer










          answered Nov 20 at 13:33









          Emilio Pisanty

          81.3k21194400




          81.3k21194400






















              up vote
              2
              down vote













              Here's the way to think about this -- why is the standard Euclidean dot product, $sum x_iy_i$ interesting? Well, it is interesting primarily from the perspective of rotations, due to the fact that rotations leave dot products invariant. The reason this is so is that this dot product can be written as $|x||y|cosDeltatheta$, and rotations leave magnitudes and relative angles invariant.



              Is the standard Euclidean norm $|x|$ invariant under Lorentz transformations? Of course not -- for instance, $Delta t^2+Delta x^2$ is clearly not invariant, but $Delta t^2-Delta x^2$ is. Similarly, $E^2+p^2$ is not important, but $E^2-p^2$ is. The reason this is the case is that Lorentz boosts are fundamentally skew transformations, which means the invariant locus is a hyperbola, not a circle. So you have $cosh^2 xi - sinh^2 xi = 1$, and $x_0^2-x_1^2$ is the right way to think of the norm on Minkowski space.



              Similarly, Lorentz boosts change the rapidity $xi$ by a simple displacement, so $Delta xi$ is invariant. From this point, it's a simple exercise to show that



              $$|x||y|coshxi=x_0y_0-x_1y_1$$



              (as for the remaining dimensions -- remember that the standard Euclidean dot product is still relevant in space, so you just need to write $x_0y_0-xcdot y=x_0y_0-x_1y_1-x_2y_2-x_3y_3$.)






              share|cite|improve this answer

























                up vote
                2
                down vote













                Here's the way to think about this -- why is the standard Euclidean dot product, $sum x_iy_i$ interesting? Well, it is interesting primarily from the perspective of rotations, due to the fact that rotations leave dot products invariant. The reason this is so is that this dot product can be written as $|x||y|cosDeltatheta$, and rotations leave magnitudes and relative angles invariant.



                Is the standard Euclidean norm $|x|$ invariant under Lorentz transformations? Of course not -- for instance, $Delta t^2+Delta x^2$ is clearly not invariant, but $Delta t^2-Delta x^2$ is. Similarly, $E^2+p^2$ is not important, but $E^2-p^2$ is. The reason this is the case is that Lorentz boosts are fundamentally skew transformations, which means the invariant locus is a hyperbola, not a circle. So you have $cosh^2 xi - sinh^2 xi = 1$, and $x_0^2-x_1^2$ is the right way to think of the norm on Minkowski space.



                Similarly, Lorentz boosts change the rapidity $xi$ by a simple displacement, so $Delta xi$ is invariant. From this point, it's a simple exercise to show that



                $$|x||y|coshxi=x_0y_0-x_1y_1$$



                (as for the remaining dimensions -- remember that the standard Euclidean dot product is still relevant in space, so you just need to write $x_0y_0-xcdot y=x_0y_0-x_1y_1-x_2y_2-x_3y_3$.)






                share|cite|improve this answer























                  up vote
                  2
                  down vote










                  up vote
                  2
                  down vote









                  Here's the way to think about this -- why is the standard Euclidean dot product, $sum x_iy_i$ interesting? Well, it is interesting primarily from the perspective of rotations, due to the fact that rotations leave dot products invariant. The reason this is so is that this dot product can be written as $|x||y|cosDeltatheta$, and rotations leave magnitudes and relative angles invariant.



                  Is the standard Euclidean norm $|x|$ invariant under Lorentz transformations? Of course not -- for instance, $Delta t^2+Delta x^2$ is clearly not invariant, but $Delta t^2-Delta x^2$ is. Similarly, $E^2+p^2$ is not important, but $E^2-p^2$ is. The reason this is the case is that Lorentz boosts are fundamentally skew transformations, which means the invariant locus is a hyperbola, not a circle. So you have $cosh^2 xi - sinh^2 xi = 1$, and $x_0^2-x_1^2$ is the right way to think of the norm on Minkowski space.



                  Similarly, Lorentz boosts change the rapidity $xi$ by a simple displacement, so $Delta xi$ is invariant. From this point, it's a simple exercise to show that



                  $$|x||y|coshxi=x_0y_0-x_1y_1$$



                  (as for the remaining dimensions -- remember that the standard Euclidean dot product is still relevant in space, so you just need to write $x_0y_0-xcdot y=x_0y_0-x_1y_1-x_2y_2-x_3y_3$.)






                  share|cite|improve this answer












                  Here's the way to think about this -- why is the standard Euclidean dot product, $sum x_iy_i$ interesting? Well, it is interesting primarily from the perspective of rotations, due to the fact that rotations leave dot products invariant. The reason this is so is that this dot product can be written as $|x||y|cosDeltatheta$, and rotations leave magnitudes and relative angles invariant.



                  Is the standard Euclidean norm $|x|$ invariant under Lorentz transformations? Of course not -- for instance, $Delta t^2+Delta x^2$ is clearly not invariant, but $Delta t^2-Delta x^2$ is. Similarly, $E^2+p^2$ is not important, but $E^2-p^2$ is. The reason this is the case is that Lorentz boosts are fundamentally skew transformations, which means the invariant locus is a hyperbola, not a circle. So you have $cosh^2 xi - sinh^2 xi = 1$, and $x_0^2-x_1^2$ is the right way to think of the norm on Minkowski space.



                  Similarly, Lorentz boosts change the rapidity $xi$ by a simple displacement, so $Delta xi$ is invariant. From this point, it's a simple exercise to show that



                  $$|x||y|coshxi=x_0y_0-x_1y_1$$



                  (as for the remaining dimensions -- remember that the standard Euclidean dot product is still relevant in space, so you just need to write $x_0y_0-xcdot y=x_0y_0-x_1y_1-x_2y_2-x_3y_3$.)







                  share|cite|improve this answer












                  share|cite|improve this answer



                  share|cite|improve this answer










                  answered Nov 20 at 15:59









                  Abhimanyu Pallavi Sudhir

                  4,41642343




                  4,41642343






















                      up vote
                      0
                      down vote













                      A vector $mathbf{v} = v^i , mathbf{e}_i = q^j ,mathbf{u}_j$ has different vector components ($v^i$, $q^j$ in this case) in different bases (${mathbf{e}}$,${mathbf{u}}$, in our example) which we can interpret as different reference frames (different axes with different origins).



                      Physicists are lazy: they refer to the vector components $v^i$ as vectors, which is a misnomer! A true vector $mathbf{v}$ exists out width whichever basis you choose to work in but to know its entries you must reference these with respect to a given basis: this is just elementary linear algebra.



                      Now, the magnitude of a vector is independent of whichever bases you choose for its description (that is, geometrically speaking its length is fixed):



                      $$
                      v^2 = v^i ; v_i , (mathbf{e}^i cdot mathbf{e}_i) = q^{j} , q_j,(mathbf{u}_jcdot mathbf{u}^j)
                      .
                      tag{assuming orthonormal bases}
                      $$



                      Hence, scalars do not transform upon of change of basis. In fact it doesn't make much sense to talk about basis for scalars since intuitively these are just numbers.



                      However, one other way to look at this is to consider a scalar a special type of vector with only one entry and one orthonormal basis (the number 1): its "length" must also be fixed. Hence, this one-dimensional "vector" is the same independent of reference frame.



                      This is true for all vectors, including special relativistic four-vectors.



                      As a sanity check, one of the tenets of special relativity is that $c$, the speed of light and a scalar, is the same for all observers. This could not be so if it was somehow different in different frames.






                      share|cite|improve this answer























                      • @EmilioPisanty Yes, indeed! Fixed.
                        – OldTomMorris
                        Nov 20 at 21:59















                      up vote
                      0
                      down vote













                      A vector $mathbf{v} = v^i , mathbf{e}_i = q^j ,mathbf{u}_j$ has different vector components ($v^i$, $q^j$ in this case) in different bases (${mathbf{e}}$,${mathbf{u}}$, in our example) which we can interpret as different reference frames (different axes with different origins).



                      Physicists are lazy: they refer to the vector components $v^i$ as vectors, which is a misnomer! A true vector $mathbf{v}$ exists out width whichever basis you choose to work in but to know its entries you must reference these with respect to a given basis: this is just elementary linear algebra.



                      Now, the magnitude of a vector is independent of whichever bases you choose for its description (that is, geometrically speaking its length is fixed):



                      $$
                      v^2 = v^i ; v_i , (mathbf{e}^i cdot mathbf{e}_i) = q^{j} , q_j,(mathbf{u}_jcdot mathbf{u}^j)
                      .
                      tag{assuming orthonormal bases}
                      $$



                      Hence, scalars do not transform upon of change of basis. In fact it doesn't make much sense to talk about basis for scalars since intuitively these are just numbers.



                      However, one other way to look at this is to consider a scalar a special type of vector with only one entry and one orthonormal basis (the number 1): its "length" must also be fixed. Hence, this one-dimensional "vector" is the same independent of reference frame.



                      This is true for all vectors, including special relativistic four-vectors.



                      As a sanity check, one of the tenets of special relativity is that $c$, the speed of light and a scalar, is the same for all observers. This could not be so if it was somehow different in different frames.






                      share|cite|improve this answer























                      • @EmilioPisanty Yes, indeed! Fixed.
                        – OldTomMorris
                        Nov 20 at 21:59













                      up vote
                      0
                      down vote










                      up vote
                      0
                      down vote









                      A vector $mathbf{v} = v^i , mathbf{e}_i = q^j ,mathbf{u}_j$ has different vector components ($v^i$, $q^j$ in this case) in different bases (${mathbf{e}}$,${mathbf{u}}$, in our example) which we can interpret as different reference frames (different axes with different origins).



                      Physicists are lazy: they refer to the vector components $v^i$ as vectors, which is a misnomer! A true vector $mathbf{v}$ exists out width whichever basis you choose to work in but to know its entries you must reference these with respect to a given basis: this is just elementary linear algebra.



                      Now, the magnitude of a vector is independent of whichever bases you choose for its description (that is, geometrically speaking its length is fixed):



                      $$
                      v^2 = v^i ; v_i , (mathbf{e}^i cdot mathbf{e}_i) = q^{j} , q_j,(mathbf{u}_jcdot mathbf{u}^j)
                      .
                      tag{assuming orthonormal bases}
                      $$



                      Hence, scalars do not transform upon of change of basis. In fact it doesn't make much sense to talk about basis for scalars since intuitively these are just numbers.



                      However, one other way to look at this is to consider a scalar a special type of vector with only one entry and one orthonormal basis (the number 1): its "length" must also be fixed. Hence, this one-dimensional "vector" is the same independent of reference frame.



                      This is true for all vectors, including special relativistic four-vectors.



                      As a sanity check, one of the tenets of special relativity is that $c$, the speed of light and a scalar, is the same for all observers. This could not be so if it was somehow different in different frames.






                      share|cite|improve this answer














                      A vector $mathbf{v} = v^i , mathbf{e}_i = q^j ,mathbf{u}_j$ has different vector components ($v^i$, $q^j$ in this case) in different bases (${mathbf{e}}$,${mathbf{u}}$, in our example) which we can interpret as different reference frames (different axes with different origins).



                      Physicists are lazy: they refer to the vector components $v^i$ as vectors, which is a misnomer! A true vector $mathbf{v}$ exists out width whichever basis you choose to work in but to know its entries you must reference these with respect to a given basis: this is just elementary linear algebra.



                      Now, the magnitude of a vector is independent of whichever bases you choose for its description (that is, geometrically speaking its length is fixed):



                      $$
                      v^2 = v^i ; v_i , (mathbf{e}^i cdot mathbf{e}_i) = q^{j} , q_j,(mathbf{u}_jcdot mathbf{u}^j)
                      .
                      tag{assuming orthonormal bases}
                      $$



                      Hence, scalars do not transform upon of change of basis. In fact it doesn't make much sense to talk about basis for scalars since intuitively these are just numbers.



                      However, one other way to look at this is to consider a scalar a special type of vector with only one entry and one orthonormal basis (the number 1): its "length" must also be fixed. Hence, this one-dimensional "vector" is the same independent of reference frame.



                      This is true for all vectors, including special relativistic four-vectors.



                      As a sanity check, one of the tenets of special relativity is that $c$, the speed of light and a scalar, is the same for all observers. This could not be so if it was somehow different in different frames.







                      share|cite|improve this answer














                      share|cite|improve this answer



                      share|cite|improve this answer








                      edited Nov 20 at 21:58

























                      answered Nov 20 at 15:33









                      OldTomMorris

                      11




                      11












                      • @EmilioPisanty Yes, indeed! Fixed.
                        – OldTomMorris
                        Nov 20 at 21:59


















                      • @EmilioPisanty Yes, indeed! Fixed.
                        – OldTomMorris
                        Nov 20 at 21:59
















                      @EmilioPisanty Yes, indeed! Fixed.
                      – OldTomMorris
                      Nov 20 at 21:59




                      @EmilioPisanty Yes, indeed! Fixed.
                      – OldTomMorris
                      Nov 20 at 21:59


















                      draft saved

                      draft discarded




















































                      Thanks for contributing an answer to Physics Stack Exchange!


                      • Please be sure to answer the question. Provide details and share your research!

                      But avoid



                      • Asking for help, clarification, or responding to other answers.

                      • Making statements based on opinion; back them up with references or personal experience.


                      Use MathJax to format equations. MathJax reference.


                      To learn more, see our tips on writing great answers.





                      Some of your past answers have not been well-received, and you're in danger of being blocked from answering.


                      Please pay close attention to the following guidance:


                      • Please be sure to answer the question. Provide details and share your research!

                      But avoid



                      • Asking for help, clarification, or responding to other answers.

                      • Making statements based on opinion; back them up with references or personal experience.


                      To learn more, see our tips on writing great answers.




                      draft saved


                      draft discarded














                      StackExchange.ready(
                      function () {
                      StackExchange.openid.initPostLogin('.new-post-login', 'https%3a%2f%2fphysics.stackexchange.com%2fquestions%2f442119%2fwhy-is-the-scalar-product-of-two-four-vectors-lorentz-invariant%23new-answer', 'question_page');
                      }
                      );

                      Post as a guest















                      Required, but never shown





















































                      Required, but never shown














                      Required, but never shown












                      Required, but never shown







                      Required, but never shown

































                      Required, but never shown














                      Required, but never shown












                      Required, but never shown







                      Required, but never shown







                      Popular posts from this blog

                      QoS: MAC-Priority for clients behind a repeater

                      Ивакино (Тотемский район)

                      Can't locate Autom4te/ChannelDefs.pm in @INC (when it definitely is there)